Translate

Saturday, June 7, 2014

Định lí thặng dư Trung Hoa và ứng dụng

Bài toán 1 : Chứng minh rằng với mọi số nguyên dương n, luôn tồn tại một tập hợp S gồm n phần tử sao cho bất kì một tập con nào của S cũng có tổng các phần tử là lũy thừa của một số tự nhiên.
Lời giải :
Gỉa sử rằng :
S=\left \{ a_1,a_2,...,a_{n-1},a_n \right \}
Ta chọn các số a_i như sau :
a_1=1^{b_1+1}2^{b_2}3^{b_3}...k^{b_k}
a_2=1^{b_1}2^{b_2+1}3^{b_3}...k^{b_k}
...
a_i=1^{b_1}2^{b_2}...(i-1)^{b_{i-1}}.i^{b_i+1}.(i+1)^{b_{i+1}}...k^{b_k}
...
a_n=1^{b_1}2^{b_2}3^{b_3}...n^{b_n+1}...k^{b_k}
Trong đó k là số nguyên dương thỏa mãn k>1+2+3+...+n.
Gỉa sử T=\left \{ a_{i_1},a_{i_2},...a_{i_m} \right \}\subset S (m<n).
Khi đó ta có :
a_{i_1}+a_{i_2}+...+a_{i_m}=1^{b_1}2^{b_2}...i_1^{b_{i_1}+1}...k^{b_k}+1^{b_1}2^{b_2}...i_2^{b_{i_2}+1}...k^{b_k}+...+1^{b_1}2^{b_2}...i_m^{b_{i_m}+1}...k^{b_k}=1^{b_1}2^{b_2}...k^{b_k}(i_1+i_2+...+i_m)=1^{b_1}2^{b_2}...(i_1+i_2+...+i_m)^{b_{i_1+i_2+...+i_m}}...k^{b_k}
Ta sẽ chọn ra k số nguyên tố phân biệt p_1,p_2,...,p_k thỏa mãn hệ sau :
p_1\mid b_1+1,p_1\mid b_2,b_3,...b_k
p_2\mid b_2+1,p_2\mid b_1,b_3,...b_k
...
p_k\mid b_k+1,p_k\mid b_1,b_2,...,b_{k-1}.
Hiển nhiên chọn được theo định lí phần dư Trung Hoa, khi đó dễ thấy :
a_{i_1}+a_{i_2}+...+a_{i_m}=A^{p_{i_1+i_2+..+i_m}}
Là lũy thừa của một số tự nhiên. Trong đó :
A=1^{b_1/p_{i_1+i_2+..+i_m}}.2^{b_2/p_{i_1+i_2+...+i_m}}....(i_1+i_2+...+i_m)^{b_{i_1+i_2+...+i_m}/p_{i_1+i_2+...+i_m}}...k^{b_k/p_{i_1+i_2+...+i_m}}
và hiển nhiên A nguyên
Bài toán được giải quyết trọn vẹn.
Bài toán 2 : Chứng minh rằng nếu p_1,p_2,....,p_n là các số nguyên tố phân biệt thì phương trình x_1^{p_1}+x_2^{p_2}+....+x_{n-1}^{p_{n-1}}=x_n^{p_n} có vô số nghiệm nguyên dương (x_1,x_2,....,x_n)
Lời giải :
Ta có đẳng thức hiển nhiên sau : \underset{n-1}{\underbrace{(n-1)^k+(n-1)^k+....+(n-1)^k}}=(n-1)^{k+1}
Khi đó ta chọn x_1=(n-1)^{\frac{k}{p_1}},x_2=(n-1)^{\frac{k}{p_2}},...,x_{n-1}=(n-1)^{\frac{k}{p_{n-1}}},x_n=(n-1)^{\frac{k+1}{p_n}}
Thì ta thu được ngay phương trình x_1^{p_1}+x_2^{p_2}+....+x_{n-1}^{p_{n-1}}=x_n^{p_n}
Vậy nếu ta chỉ ra được số nguyên dương k sao cho x_1,x_2,...,x_n đều nguyên thì ta có ngay điều phải chứng minh.
Mà điều này tương đương với hệ sau có nghiệm : \left\{\begin{matrix} k\equiv 0\;(mod\;p_1)\\ k\equiv 0\;(mod\;p_2)\\ ....\\ k\equiv 0\;(mod\;p_{n-1})\\ k\equiv -1\;(mod\;p_n) \end{matrix}\right.
Điều này luôn đúng theo định lí phần dư Trung Hoa vì p_1,p_2,...,p_n là các số nguyên tố phân biệt.
Bài toán 3 : Chứng minh rằng với mọi số nguyên dương n luôn tồn tại một dãy gồm n số nguyên liên tiếp sao cho bất kì số nào trong dãy cũng đều có ước dạng 2^k-1.
Lời giải :
Bổ đề : Với m,n là các số nguyên dương và a là số nguyên dương khác 1 thì ta có gcd(a^m-1,a^n-1)=a^{gcd(m,n)}-1
Chứng minh bổ đề :
Đặt d=gcd(m,n) và \left\{\begin{matrix} m=dm'\\ n=dn' \end{matrix}\right.\;\;gcd(m',n')=1
Ta có a^m-1=a^{dm'}-1\;\vdots \;a^d-1 và a^n-1=a^{dn'}-1\;\vdots \;a^d-1
Gọi d'=gcd(a^m-1,a^n-1) thì d'\;\vdots \;a^d-1\;\;(1)
Vì d=gcd(m,n) nên theo định lí Bezout thì tồn tại hai số nguyên dương x,y sao cho mx-ny=1
Từ đó a^{mx}-1\;\vdots \;a^{m}-1\;\vdots \;d' và a^{ny}-1\;\vdots \;a^{n}-1\;\vdots \;d'
Do đó (a^{mx}-1)-(a^{ny}-1)\;\vdots \;d'\Rightarrow a^{ny}(a^{mx-ny}-1)\;\vdots\; d'\Rightarrow a^{ny}(a^d-1)\;\vdots \;d'
Nhưng vì a^{ny}-1\;\vdots d'\;\Rightarrow d'\nmid a^{ny}-1
Nên a^{d}-1\;\vdots \;d'\;\;(2)
Từ (1)(2) suy ra a^d-1=d'.
Bổ đề chứng minh hoàn tất.
Trở lại bài toán :
Xét hệ đồng dư tuyến tính :
\left\{\begin{matrix} x\equiv -1\;\;(mod\;2^{p_1}-1)\\ x\equiv -2\;\;(mod\;2^{p_2}-1)\\ ....\\ x\equiv -n\;\;(mod\;2^{p_n}-1) \end{matrix}\right.
Với p_1,p_2,...,p_n là các số nguyên tố phân biệt
Theo bổ đề ta có gcd(2^{p_i}-1,2^{p_j-1})=2^{gcd(p_i,p_j)}-1=2-1=1 với mọi i\neq j,i,j\in \left \{ 1,2,...,n \right \}
Từ đó theo định lí phần dư Trung Hoa thì hệ này có nghiệm.
Suy ra điều cần chứng minh.
Bài toán 4 : Chứng minh rằng với mọi số nguyên dương n luôn tồn tại n số nguyên a_1,a_2,...,a_n sao cho a_i+a_j  là lũy thừa của một số tự nhiên với số mũ lớn hơn 1 với mọi i,j\in \left \{ 1,2,3,...,n \right \}
Lời giải :
Ta chọn các số như sau :
\left\{\begin{matrix} a_1=1^{x_1+1}2^{x_2}3^{x_3}...(2n)^{x_{2n}}\\ a_2=1^{x_1}2^{x_2+1}3^{x_3}...(2n)^{x_{2n}}\\ ...\\ a_n=1^{x_1}2^{x_2}3^{x_3}...n^{x_n+1}..(2n)^{x_{2n}} \end{matrix}\right.
Khi đó thì :
a_i+a_j=1^{x_1}2^{x_2}...i^{x_i+1}...(2n)^{x_{2n}}+1^{x_1}2^{x_2}...j^{x_j+1}...(2n)^{x_{2n}}=1^{x_1}2^{x_2}...(2n)^{x_{2n}}(i+j)=1^{x_1}2^{x_2}...(i+j)^{x_{i+j}+1}...(2n)^{x_{2n}}
Xét các số nguyên tố p_1,p_2,...,p_{2n} phân biệt
Xét các hệ đồng dư tuyến tính :
\left\{\begin{matrix} x_1\equiv -1\;(mod\;p_1)\\ x_1\equiv 0\;(mod\;p_k)\forall k\in \left \{ 2,3,...,2n-1,2n \right \} \end{matrix}\right.
\left\{\begin{matrix} x_2\equiv -1\;(mod\;p_2)\\ x_2\equiv 0\;(mod\;p_k)\forall k\in \left \{ 1,3,4...,2n \right \} \end{matrix}\right.
….
\left\{\begin{matrix} x_{i+j}\equiv -1\;(mod\;p_{i+j})\\ x_{i+j}\equiv 0\;(mod\;p_k)\forall k\in \left \{ 1,2,...,i+j-1,i+j+1,...,2n \right \} \end{matrix}\right.
….
\left\{\begin{matrix} x_{2n}\equiv -1\;(mod\;p_{2n})\\ x_{2n}\equiv 0\;(mod\;p_k)\forall k\in \left \{ 1,2,...,2n-1 \right \} \end{matrix}\right.
Theo định lí phần dư Trung Hoa thì các hệ này chắc chắn có nghiệm
Từ đó suy ra :
\dfrac{x_1}{p_1},\dfrac{x_2}{p_2},...,\dfrac{x_k+1}{p_k},...,\dfrac{x_{2n}}{p_{2n}}\in \mathbb{Z}\;\forall k=\overline{1,2n}
Khi đó a_i+a_j=1^{x_1}2^{x_2}...(i+j)^{x_{i+j}+1}..(2n)^{x_{2n}}=\left [ 1^{\frac{x_1}{p_{i+j}}}.2^{\frac{x_2}{p_{i+j}}}...(i+j)^{\frac{x_{i+j}+1}{p_{i+j}}} ...(2n)^{\frac{x_{2n}}{p_{i+j}}}\right ]^{p_{i+j}} là lũy thừa của một số tự nhiên.
Đây là điều phải chứng minh
Bài toán 5 : Cho p là số nguyên tố. Chứng minh rằng tồn tại một bội số của p sao cho 10 chữ số tận cùng của nó đôi một khác nhau.
Lời giải :
Nếu p=2 thì hiển nhiên luôn tồn tại một số thỏa đề, ví dụ 1234567899876543210
Nếu p=5 thì cũng luôn tồn tại một số thỏa đề, ví dụ 1234567899876432105
Xét p\notin \left \{ 2,5 \right \}.
Xét hệ đồng dư tuyến tính : \left\{\begin{matrix} x\equiv \overline{a_0a_1a_2...a_{9}}\;(mod\;10^{10})\\ x\equiv 0\;(mod\;p) \end{matrix}\right. trong đó a_i đôi một khác nhau và a_{i}\in \left \{ 0,1,2,3,4,5,6,7,8,9 \right \}.
Vì p\in \mathbb{P},p\neq 2,p\neq 5 nên gcd(p,10^{10})=1, do đó theo định lí phần dư Trung Hoa thì hệ này chắc chắn có nghiệm, nghiệm của hệ chính là số thỏa mãn đề bài.
Ta có điều phải chứng minh.
Bài toán 6  (Balkan 2000) : Cho tập A=\left \{ a_1,a_2,...,a_k \right \} với a_i\in \mathbb{N}\;\forall i=\overline{1,k}. Chứng minh rằng tồn tại số nguyên n sao cho các phần tử của tập B=\left \{ na_1,na_2,...,na_k \right \} đều là lũy thừa của một số tự nhiên với số mũ lớn hơn 1.
Lời giải :
Xét k số nguyên tố phân biệt p_1,p_2,...,p_k.
Xét các hệ đồng dư tuyến tính :
\left\{\begin{matrix} x_1\equiv -1\;(mod\;p_1)\\ x_1\equiv 0\;(mod\;p_i)\;\;\forall i\in \left \{ 2,3,..,k \right \}\\\ \end{matrix}\right.
\left\{\begin{matrix} x_2\equiv -1\;(mod\;p_2)\\ x_2\equiv 0\;(mod\;p_i)\;\;\forall i\in \left \{ 1,3,..,k \right \}\\\ \end{matrix}\right.
\left\{\begin{matrix} x_3\equiv -1\;(mod\;p_3)\\ x_3\equiv 0\;(mod\;p_i)\;\;\forall i\in \left \{ 1,2,4,..,k \right \}\\\ \end{matrix}\right.
….
\left\{\begin{matrix} x_k\equiv -1\;(mod\;p_k)\\ x_k\equiv 0\;(mod\;p_i)\;\;\forall i\in \left \{ 1,2,3,..,k-1 \right \}\\\ \end{matrix}\right.
Theo định lí phần dư Trung Hoa thì các hệ này đều có nghiệm.
Từ đó ta suy ra rằng \left\{\begin{matrix} \dfrac{x_1+1}{p_1},\dfrac{x_2}{p_1},...,\dfrac{x_k}{p_1}\in \mathbb{Z}\\ \\\ \dfrac{x_2}{p_1},\dfrac{x_2+1}{p_2},...,\dfrac{x_2}{p_k}\in \mathbb{Z}\\ .\\ .\\ \dfrac{x_k}{p_1},\dfrac{x_k}{p_2},...,\dfrac{x_k+1}{p_k}\in \mathbb{Z} \end{matrix}\right.
Khi đó chọn số n=a_1^{x_1}a_2^{x_2}...a_k^{x_k} thì na_1=a_1^{x_1+1}a_2^{x_2}...a_k^{x_k}=\left ( a_1^{\frac{x_1+1}{p_1}}a_2^{\frac{x_2}{p_1}}...a_k^{\frac{x_k}{p_1}} \right )^{p_1}na_2=a_1^{x_1}a_2^{x_2+1}...a_k^{x_k}=\left ( a_1^{\frac{x_1}{p_2}}a_2^{\frac{x_2+1}{p_2}}...a_k^{\frac{x_k}{p_2}} \right )^{p_2}, ….,na_k=a_1^{x_1}a_2^{x_2}...a_k^{x_k+1}=\left ( a_1^{\frac{x_1}{p_k}}a_2^{\frac{x_2}{p_k}}...a_k^{\frac{x_k+1}{p_k}} \right )^{p_k}
Điều này cho thấy các phần tử của B đều là lũy thừa của một số tự nhiên.
Suy ra điều phải chứng minh.
Bài toán 7 : Cho hai số nguyên dương p,q nguyên tố cùng nhau. Chứng minh rằng tồn tại số nguyên ksao cho (pq-1)^n.k+1 là hợp số với mọi số nguyên dương n.
Lời giải :
Xét hệ đồng dư \left\{\begin{matrix} k\equiv 1\;(mod\;p)\\ k\equiv -1\;(mod\;q) \end{matrix}\right., do gcd(p,q)=1 nên theo định lí phần dư Trung Hoa thì hệ này chắc chắn có nghiệm.
Nếu n chẵn thì (pq-1)^n.k+1\equiv k+1\equiv -1+1=0\;\;(mod\;q), suy ra (pq-1)^n.k+1 là hợp số
Nếu n lẻ thì (pq-1)^n.k+1\equiv -k+1\equiv -1+1=0\;\;(mod\;p), suy ra (pq-1)^n.k+1 là hợp số.
Kết luận : Luôn tồn tại số k sao cho (pq-1)^n.k+1 là hợp số với mọi số nguyên dương n.
Bài toán 8:
a) Chứng minh rằng với mọi số nguyên dương n bất kì, luôn tồn tại n số nguyên dương liên tiếp mà trong đó không có số nào là lũy thừa của một số nguyên tố.
b) Chứng minh rằng với mọi số nguyên dương n tồn tại n số nguyên dương liên tiếp sao cho bất kì số nào trong chúng cũng chia hết cho n số nguyên tố liên tiếp.
Lời giải :
a) Xét hệ đồng dư tuyến tính :
\left\{\begin{matrix} x\equiv -1\;(mod\;p_1p_2)\\ x\equiv -2\;(mod\;p_3p_4)\\ x\equiv -3\;(mod\;p_5p_6)\\ ...\\ x\equiv -n\;(mod\;p_{2n-1}p_{2n}) \end{matrix}\right.
Trong đó p_{1},p_{2},...,p_{2n} là các số nguyên tố phân biệt.
Dễ dàng thấy rằng theo định lí phần dư Trung Hoa, hệ này chắc chắn có nghiệm x_0.
Khi đó, ta có p_1p_2\mid x_{0}+1,p_3p_4\mid x_0+2,...,p_{2n-1}p_{2n}\mid x_0+n, như vậy dãy x_{0}+1,x_0+2,..,x_0+n gồm n số nguyên dương liên tiếp mà trong đó không có số nào là lũy thừa của một số nguyên tố (điều phải chứng minh)
b) Xét hệ đồng dư tuyến tính :
\left\{\begin{matrix} x\equiv -1\;(mod\;p_1p_2...p_n)\\ x\equiv -2\;(mod\;p_{n+1}p_{n+2}...p_{2n})\\ x\equiv -3\;(mod\;p_{2n+1}p_{2n+2}...p_{3n})\\ ...\\ x\equiv -n\;(mod\;p_{n^2-n+1}p_{n^2-n+2}...p_{n^2}) \end{matrix}\right.
Trong đó p_{i}\in \mathbb{P},\forall i=1,2,...,n^2 với kí hiệu p_{i},p_{i+1} được coi là hai số nguyên tố liên tiếp.
Theo định lí phần dư Trung Hoa thì hệ này chắc chắn có nghiệm x_0, khi đó dãy x_0+1,x_0+2,...,x_0+n có n số nguyên dương liên tiếp mà trong đó số nào cũng chia hết cho nsố nguyên tố liên tiếp (điều phải chứng minh)
Bài toán 9 : Tìm tất cả các số nguyên dương n sao cho luôn tồn tại số nguyên m thỏa mãn 2^n-1\mid m^2+9.
Lời giải :
Bổ đề 1 : Một số nguyên có dạng 4k+3 thì luôn tồn tại một ước số nguyên tố p\equiv 3\;(mod\;4)
Chứng minh bổ đề 1 :
Số nguyên a dạng 4k+3 là một số lẻ nên nó không có ước nguyên tố 2.
Gỉa sử a=p_{1}^{\alpha _1}p_{2}^{\alpha _{2}}...p_{n}^{\alpha _n}\;\;\;(p_i\in \mathbb{P},\alpha _i\in \mathbb{N}^*,i=\overline{1,n})
Nếu p_i\equiv 4\;(mod\;4)\;\forall i=1,2,...,n\Rightarrow a\equiv 1\;(mod\;4) , mâu thuẫn với giả thiết.
Do đó a có ít nhất một ước nguyên tố p\equiv 3\;(mod\;4)
Bổ đề 2 : Nếu các số nguyên x,y và số nguyên tố p\equiv 3\;(mod\;4) thỏa mãn p\mid x^2+y^2 thì p\mid x,p\mid y.
Chứng minh bổ đề 2 :
Nếu p|x hoặc p|y thì hiển nhiên ta có điều phải chứng minh. Xét p\nmid x,p\nmid y
Đặt p=4k+3\qquad(k\in \mathbb{N})
Theo định lí Fermat nhỏ : x^{4k+2}+y^{4k+2}\equiv x^{p-1}+y^{p-1}\equiv 2\;(mod\;p)\qquad(1)
Mặt khác x^{2}+y^2\equiv 0\;(mod\;p)\Rightarrow x^{4k+2}+y^{4k+2}\equiv 0\;(mod\;p)\qquad(2)
Từ (1)(2) suy ra p=2, mâu thuẫn.
Trở lại bài toán :
Nếu n=1 thì hiển nhiên với mọi m nguyên, bài toán đều được thỏa mãn.
Xét n>1. Gọi q là một ước nguyên tố lẻ của n, khi đó 2^{q}-1\equiv 3\;(mod\;4) nên 2^q-1 có ít nhất một ước nguyên tố p\equiv 3\;(mod\;4). Khi đó dễ dàng thấy rằng :
m^2+9\;\vdots \;2^{n}-1\;\vdots \;2^q-1\;\vdots\; p
Khi đó theo bổ đề 2 ta có p|3,p=3. Suy ra 3\mid 2^{q}-1\Rightarrow 2\mid q, điều này mâu thuẫn vì q lẻ.
Như vậy n không có ước nguyên tố lẻ, do đó n=2^k\;(t\in \mathbb{N}^{*}).
Ta sẽ chứng minh rằng với mọi t\in \mathbb{N}^{*} thì n=2^t luôn thỏa mãn đề bài.
Thật vậy,
Ta có 2^n-1=2^{2^{k}}-1=(2^{2^0}+1)(2^{2^1}+1)(2^{2^2}+1)...(2^{2^{t-1}}+1)
Xét hệ đồng dư tuyến tính :
\left\{\begin{matrix} x\equiv 0\;(mod\;2^{2^0}+1)\\ x\equiv 3.2^{2^{0}}\;(mod\;2^{2^1}+1)\\ x\equiv 3.2^{2^{1}}\;(mod\;2^{2^{2}}+1)\\ ...\\ x\equiv 3.2^{2^{k-2}}\;(mod\;2^{2^{k-1}}+1) \end{matrix}\right.
Dễ thấy rằng gcd(2^{2^i}+1,2^{2^j}+1)=1\;\forall i\neq ,j,i,j\in \left \{ 0,1,2,...,k-1 \right \} vì chúng là các số Fermat.
Theo định lí phần dư Trung Hoa thì hệ này chắc chắn có nghiệm x_0.
Ta suy ra
\left\{\begin{matrix} x_{0}^{2}\equiv 0\equiv -9\;(mod\;2^{2^0}+1)\\ x_0^2\equiv 9.2^{2^{1}}\equiv -9\;(mod\;2^{2^1}+1)\\ x_0^2\equiv 9.2^{2^2}\equiv -9\;(mod\;2^{2^2}+1)\\ ...\\ x_0^2\equiv 9.2^{2^{k-1}}\equiv -9\;(mod\;2^{2^{k-1}}+1) \end{matrix}\right.\Rightarrow x_0^2+9\equiv 0\;(mod\;2^n-1)
Khi đó với mọi n=2^k thì luôn tồn tại số nguyên m=x_0 là nghiệm của hệ trên thỏa mãn đề bài.
Kết luận : \boxed{n=2^k\;\;(k\in \mathbb{N})}

1 comment:

Unknown said...
This comment has been removed by the author.